Truth of Mathematical Statements: Which is Correct?

In summary: Conversely, the first statement is false if you take ##<## to be an irreflexive order.In summary, the second statement is true since for all x greater than 0, there exists at least one value of y > 0 such that y <x. However, the first statement can also be true depending on the universe of discourse and the interpretation of the symbol <.
  • #1
HMPARTICLE
95
0
Which one of these statements is true?

$$ \exists y >0 : \forall x > 0, y < x $$

or

$$ \forall x > 0 \exists y > 0 : y < x $$I think the second statement is correct, since for all x greater than 0, there exists at least one value of y > 0 such that y <x.

The first statement doesn't really make a lot of sense, there exists at least one value of y >0 such that for all x >0, y < x. What this says to me is that there are values of y which are less than all the values of x > 0. which can't be true since that would imply $$ y \le 0 $$

Could someone tell my why i am correct, or why i am wrong. Please!
 
Physics news on Phys.org
  • #2
You are right.
The real numbers should allow you to find a number smaller than any fixed positive number. So whichever one you fix, it cannot be smaller (or larger) than all others. There will always exist one smaller or larger.
 
  • #3
HMPARTICLE said:
Which one of these statements is true?

$$ \exists y >0 : \forall x > 0, y < x $$

or

$$ \forall x > 0 \exists y > 0 : y < x $$I think the second statement is correct, since for all x greater than 0, there exists at least one value of y > 0 such that y <x.

The first statement doesn't really make a lot of sense, there exists at least one value of y >0 such that for all x >0, y < x. What this says to me is that there are values of y which are less than all the values of x > 0. which can't be true since that would imply $$ y \le 0 $$

Could someone tell my why i am correct, or why i am wrong. Please!

If the point of the exercise is to understand the quantifiers and how their order matters, then I would say that your assessment is correct. To be fair, your "explanation" for why the second statement is true is really just a translation of the symbols. You should try to come up with a concrete example of a ##y## satisfying ##0<y<x##.

However ...

In order to truly say whether the statements are true or false, you would first need to stipulate the universe of discourse. For instance, the second statement is true if ##x## and ##y## are real or rational variables and ##<## is the standard ordering and false if they are integer or natural number variables with the standard ordering.

The first statement can be true if you adopt a slightly nonstandard interpretation for ##<## in the right universe; e.g. take ##<## to be a reflexive order so that ##\forall x:x<x## is true.
 

Related to Truth of Mathematical Statements: Which is Correct?

1. What is the definition of a mathematical statement?

A mathematical statement is a sentence or expression that can be proven to be either true or false using mathematical reasoning and logical principles.

2. How do you determine if a mathematical statement is true or false?

A mathematical statement is considered true if it is supported by mathematical principles and can be logically proven to be true. It is considered false if it can be disproven or contradicted by mathematical principles.

3. Can a mathematical statement be both true and false?

No, a mathematical statement cannot be both true and false. It can only have one truth value, either true or false, based on mathematical principles and logical reasoning.

4. Are all mathematical statements objective truths?

Yes, all mathematical statements are considered objective truths because they are based on mathematical principles and logical reasoning, rather than personal opinions or beliefs.

5. Can a mathematical statement be proven to be true or false?

Yes, a mathematical statement can be proven to be true or false using mathematical reasoning and logical principles. However, some statements may require more complex proofs and may still be considered unproven.

Similar threads

  • Calculus and Beyond Homework Help
Replies
5
Views
892
  • Calculus and Beyond Homework Help
Replies
2
Views
879
  • Calculus and Beyond Homework Help
Replies
20
Views
2K
  • Calculus and Beyond Homework Help
Replies
20
Views
2K
  • Calculus and Beyond Homework Help
Replies
8
Views
488
  • Calculus and Beyond Homework Help
Replies
2
Views
1K
  • Calculus and Beyond Homework Help
Replies
6
Views
1K
  • Calculus and Beyond Homework Help
Replies
3
Views
533
  • Calculus and Beyond Homework Help
Replies
3
Views
823
  • Calculus and Beyond Homework Help
Replies
11
Views
1K
Back
Top